If both photographs in one of the three sections are by Gagnon, then which one of the following could be true?

toyalli on February 1, 2020

Could not understand why D is incorrect

Please help

Replies
Create a free account to read and take part in forum discussions.

Already have an account? log in

Skylar on February 1, 2020

@toyalli, happy to help!

Ultimately, answer choice (D) is incorrect because of Rule #3.

(D) states that section M consists of G and H. If this were the case, section L would have to consist of G and G, since Rule #4 tells us that G cannot go in section S and the question setup tells us that we need a double G in one of the sections. Rule #1 tells us that a photographer's work can be used a minimum of one time and a maximum of three times. This leaves F and either F or H to fill in section S, giving us:
L: G G
M: G H
S: F (F/H)

However, Rule #3 states that that the number of Hue's photos in L has to be equal to the number of Fuentes' photos in S. In the above scenario, we have to have at least one of Fuentes' photos in S, but we don't have any of Hue's in L, so the numbers are unequal. This violates Rule #3 and makes (D) incorrect.

Does that make sense? Please let us know if you have any other questions!

toyalli on February 2, 2020

THANKS SO MUCH